Inequalities + Absolute Value questoin

This topic has expert replies
Junior | Next Rank: 30 Posts
Posts: 25
Joined: Mon May 04, 2009 6:38 pm
Followed by:2 members

Inequalities + Absolute Value questoin

by adt29 » Tue Jan 10, 2012 1:18 pm
Is x > 0?

(1) |x+3|<4 (2) |x-3|<4

E

I was able to get the answer but with a bit of guess work. Should one be assuming that x can be a non-integer between 0 and 1?

GMAT/MBA Expert

User avatar
GMAT Instructor
Posts: 768
Joined: Wed Dec 28, 2011 4:18 pm
Location: Berkeley, CA
Thanked: 387 times
Followed by:140 members

by Mike@Magoosh » Tue Jan 10, 2012 6:29 pm
Hi! I'm happy to contribute on this one. :)

First of all, if you are doing guesswork plugging in numbers for x, then always always consider fractions less than one as one possibility. Often those numbers behave differently from other numbers, and can be a conclusion-changer in DS.

As for this particular question, I'll do an algebraic solution in case you are interested.

Question: is x > 0

Statement #1: |x+3|<4

That means that -4 < x + 3 < 4

Subtract 3 from all three conditions, and we have: -7 < x < 1. So, under this condition, x could be negative (down to -7) or positive (between zero and 1). Remember, there's a whole infinity of numbers between zero and 1: please don't forget they're there. By itself, Statement #1 is insufficient.


Statement #2: |x-3|<4

That means that -4 < x - 3 < 4

Add three to all three conditions, and we have -1 < x < 7. So, under this condition, x can be positive (up to 7) or negative (between -1 and 0). Again, another whole infinity of numbers between -1 and zero: don't forget about them! By itself, Statement #2 is insufficient.

Combined Statements #1 & #2

We combine the conditions -7 < x < 1 and -1 < x < 7. To satisfy both of those conditions simultaneously, x must be between -1 and +1: -1 < x < 1. That's the overlap, or intersection, of the two regions. Well, x can still be positive or negative, so even combined, the statements are insufficient. Answer = E.

Does that make sense? Please let me know if you have any questions on what I've said.

Mike :)
Magoosh GMAT Instructor
https://gmat.magoosh.com/

Legendary Member
Posts: 1085
Joined: Fri Apr 15, 2011 2:33 pm
Thanked: 158 times
Followed by:21 members

7

by pemdas » Wed Jan 11, 2012 10:00 am
st(1) cases considered x+3>-4 and x+3<4, accordingly x >-7 and x<1 Insuff

st(2) cases considered x-3>-4 and x-3<4, accordingly x>-1 and x<7 Insuff

Combining st(1&2) still Insuff

answering your question: you must not assume 0<x<1, as you need to satisfy x>0 which extends beyond 1 and embraces all positive values through +ve infinity.
adt29 wrote:Is x > 0?

(1) |x+3|<4 (2) |x-3|<4




E

I was able to get the answer but with a bit of guess work. Should one be assuming that x can be a non-integer between 0 and 1?
Success doesn't come overnight!